Un problème sans problème

Bonjour,

1. ABC un triangle acutangle
2. H l'orthocentre de ABC
3. PQR le triangle orthique de ABC
4. M, N les milieux resp. de [BC], [QR],

Question : le cercle circonscrit au triangle MNH est tangent à (AP) en H.

Sincèrement
Jean-Louis

[Contenu du fichier pdf joint. AD]103970

Réponses

  • Bonjour,

    Voilà une solution en barycentriques:
    % Jean-Louis Ayme - 09/06/2020 - Un problème sans problème
    
    clc, clear all, close all
    
    syms a b c real  % Les longueurs des côtés
    
    A=[1; 0; 0]; % Sommets du triangle ABC
    B=[0; 1; 0];
    C=[0; 0; 1];
    
    BC=[1, 0, 0]; % Côtés du triangle ABC
    CA=[0, 1, 0];
    AB=[0, 0, 1];
    
    %-----------------------------------------------------------------------
    
    M = [0; 1; 1];  % Milieu de [BC]
    
    Q = ProjectionOrthogonaleBary(B,CA,a,b,c); % Pied Q de la B-hauteur
    R = ProjectionOrthogonaleBary(C,AB,a,b,c); % Pied R de la C-hauteur
    
    % On trouve
    % Q =
    % - a^2 - b^2 + c^2
    %   0
    %   a^2 - b^2 - c^2
    % R =
    % - a^2 + b^2 - c^2
    %   a^2 - b^2 - c^2
    %   0
     
    H = OrthocentreBary(A,B,C,a,b,c); % Orthocentre du triangle ABC
    
    % On trouve
    % H =
    %  (a^2 - b^2 + c^2)*(a^2 + b^2 - c^2)
    %  (- a^2 + b^2 + c^2)*(a^2 + b^2 - c^2)
    %  (- a^2 + b^2 + c^2)*(a^2 - b^2 + c^2)
     
    N = MilieuBary(Q,R,a,b,c);  % Milieu de [QR]
    
    F=8*a^2*b^2*c^2*(- a^2 + b^2 + c^2)^2; % Facteur de simplification0
    N=FactorT(N/F);
    
    % On trouve:
    % N =
    % a^2*b^2 + a^2*c^2 - b^4 + 2*b^2*c^2 - c^4
    % b^2*(- a^2 + b^2 + c^2)
    % c^2*(- a^2 + b^2 + c^2)
     
    J = CentreCercleCirconscritBary(M,N,H,a,b,c);
    
    F=(- a^2 + b^2 + c^2)*(a + b + c)*(a + b - c)*(a - b + c)*(b - a + c)/(2*b^2*c^2); % Facteur de simplification
    J=FactorT(J/F);
    
    % On trouve:
    % J =
    % 4*(b + c)*(b - c)*(a^2 - b^2 + c^2)*(a^2 + b^2 - c^2)
    % - a^6 - 4*a^4*b^2 + 4*a^4*c^2 + 3*a^2*b^4 + 2*a^2*b^2*c^2 - 5*a^2*c^4 + 2*b^6 - 2*b^4*c^2 - 2*b^2*c^4 + 2*c^6
    % a^6 - 4*a^4*b^2 + 4*a^4*c^2 + 5*a^2*b^4 - 2*a^2*b^2*c^2 - 3*a^2*c^4 - 2*b^6 + 2*b^4*c^2 + 2*b^2*c^4 - 2*c^6
    
    JH=Wedge(J,H); % Droite (JH)
    
    F = (a^6 - 3*a^2*b^4 + 6*a^2*b^2*c^2 - 3*a^2*c^4 + 2*b^6 - 2*b^4*c^2 - 2*b^2*c^4 + 2*c^6);  % Facteur de simplification
    JH=FactorT(JH/F);
    
    % On trouve:
    % JH = [ -2*a^2*(- a^2 + b^2 + c^2), (a^2 - b^2 + c^2)*(a^2 + b^2 - c^2), (a^2 - b^2 + c^2)*(a^2 + b^2 - c^2)]
    
    T=Wedge(JH,[1,0,0]); % Point d'intersection de (H) et (BC)
    
    % On trouve T = [0; 1; -1] qui est de somme nulle 
    % donc (JH) et (BC) sont parallèles
    
    Cordialement,

    Rescassol
  • Bonsoir,

    Ou un p'tit coup de Morley circonscrit:
    % Jean-Louis Ayme - 09/06/2020 - Un problème sans problème
    
    clc, clear all, close all
    
    syms a b c
    syms aB bB cB % Conjugués
    
    aB=1/a;
    bB=1/b;
    cB=1/c;
    
    syms s1 s2 s3;
    syms s1B s2B s3B; % Conjugués
    
    s1=a+b+c;
    s2=a*b+b*c+c*a;
    s3=a*b*c;
    
    s1B=s2/s3;
    s2B=s1/s3;
    s3B=1/s3;
    
    %-----------------------------------------------------------------------
    
    m=(b+c)/2;        % Milieu de [BC]
    mB=(bB+cB)/2;
    
    q=(s1*b-c*a)/(2*b); % Pied de la B-hauteur
    r=(s1*c-a*b)/(2*c); % Pied de la C-hauteur
    
    qB=(s1B*bB-cB*aB)/(2*bB); 
    rB=(s1B*cB-aB*bB)/(2*cB); 
    
    n=Factor((q+r)/2);        % Milieu de [QR]
    nB=Factor((qB+rB)/2);
    
    % On trouve n = (2*b^2*c - a*b^2 + 2*b*c^2 + 2*a*b*c - a*c^2)/(4*b*c)
    
    % Cercle circonscrit au triangle MNH
    
    [j jB] = CentreCercleCirconscrit(m,n,s1,mB,nB,s1B);
    
    j=Factor(j);
    
    % On trouve:
    
    j = (4*a^3+(b+c)*(6*a^2+a*(b+c)-2*b*c))/(4*(a^2-b*c));
    
    bc = c-b;
    bcB = cB-bB;
    
    jh = s1-j;
    jhB = s1B-jB;
    
    Nul=Factor(bc*jhB-bcB*jh) % Égal à 0, donc (JH) et (BC) sont parallèles
    
    Cordialement,

    Rescassol
  • Ou un pt'it coup d'Axiome de Pythagore!
    $$OH^2=OQ^2=OR^2=\overline{OM}.\overline{ON}\qquad$$
    Amicalement
    [small]p[/small]appus104032
  • Bonjour,

    Bien joué, Pappus.

    Cordialement,

    Rescassol
  • Bonjour,
    belle preuve pappus..
    (Juste pour rappeler que M, N et O sont sur la droite de Newton...)

    Ta preuve propose ce petit résultat : démontrer que <OQM est droit...

    Comment?

    Sincèrement
    Jean-Louis
  • Mon cher Jean-Louis
    Le coup de la droite de Newton, j'y avais pensé aussi. mais les points $M$ et $O$ sont tout simplement sur la médiatrice de $QR$.
    Cette question d'angle droit,(le seul angle qui nous reste encore très provisoirement avec l'angle plat!), est plus subtile.
    Qui va s'y coller?
    Encore moi comme d'hab?
    Amicalement
    [small]p[/small]appus
  • Mon cher Pappus,

    les cercles de diamètre [AH] et [BC] sont orthogonaux...le résultat suit...

    Pour le problème initial, les triangles HBC et HRQ étant inversement semblables...j'ai introduit le point O centre du cercle circonscrit à ABC...pour terminer, j'ai terminé avec une preuve angulaire...next on my site... pour tout mettre au propre.

    Pour terminer ce petit problème, il reste à montrer l'orthogonalité de ces deux cercles...
    Je te laisse réfléchir...une soltion différente de lla mienne peut surgir...

    Avec toutes mes amitiés
    Jean-Louis
  • Mon cher Jean-Louis
    Pour l'orthogonalité, on pourrait passer par l'évaluation de produits scalaires mais est-ce encore de la géométrie synthétique?
    On va se contenter d'une chasse aux angles orientés de droites mais qui pourrait la comprendre?
    En tout cas, on part d'un angle pour arriver inexorablement à un autre!
    $(MR,RC)=(RC,MC)\qquad$ (symétrie par rapport à la médiatrice de $RC$ dans le triangle isocèle $MRC$)
    $(RC,MC)=(CR,CP)\qquad$ (alignement des points $C$, $P$, $M$)
    $((CR,CP)=(AR,AP)\qquad$ cocyclicité des points $A$, $C$, $P$, $R$ sur le cercle de diamètre $AC$).
    $(AR,AP)=(AR,AO)\qquad$ (alignement des points $A$, $P$, $O$).
    $(AR,AO)=(RO,RA)\qquad$ (symétrie par rapport à la médiatrice de $AR$ dans le triangle isocèle $OAR$).
    Résultat des courses:
    $$(RM,RC)=(RO,RA)\qquad $$
    Comme $RC\perp RA\qquad$, cette égalité angulaire entraîne: $RM\perp RO$.
    Amicalement
    [small]p[/small]appus
  • Mon cher pappus,
    merci pour ta réponse...
    J'ai utilisé un résultat de Nathan Altshiller-Court

    Ayme J.-L., A propos de la ponctuelle (MH), G.G.G. vol. 7, p. 50-51 ; http://jl.ayme.pagesperso-orange.fr/

    Sincèrement
    jean-Louis
Connectez-vous ou Inscrivez-vous pour répondre.